LSAT and Law School Admissions Forum

Get expert LSAT preparation and law school admissions advice from PowerScore Test Preparation.

User avatar
 Dave Killoran
PowerScore Staff
  • PowerScore Staff
  • Posts: 5853
  • Joined: Mar 25, 2011
|
#43445
Complete Question Explanation
(The complete setup for this game can be found here: lsat/viewtopic.php?t=16406)

The correct answer choice is (B)

If K and N are in a group together, they create a block. F and J are also a block, and the two blocks cannot be in the same group due to lack of space. Thus, the two blocks are in separate groups, and we can reserve two more spaces in each group, even though we cannot determine exactly which variables are in each group:
D94_Game_#1_#2_diagram 1.png
With three spaces in each group reserved, we can Hurdle the Uncertainty and determine the placement of the remaining two variables, H and L. Because from the third rule if H is in group 1 then L must be in group 1, and there is insufficient room in group 1 for two more variables, we can infer that H cannot be in group 1 and therefore must be in group 2. Accordingly, answer choice (B) is correct.

To complete the diagram, as there is no more space in group 2, L must be in group 1.
You do not have the required permissions to view the files attached to this post.
 jpincus
  • Posts: 3
  • Joined: Sep 05, 2017
|
#40038
I see why B is the correct answer because it would make the groups consist of:
1: F,G,J,L
2: H,M,N,K

However, I am not seeing why choice E is wrong. If m is in group 1 couldn't it be diagrammed as:
1: N,L,H,M
2: K,F,J,G

Thank you!
User avatar
 Dave Killoran
PowerScore Staff
  • PowerScore Staff
  • Posts: 5853
  • Joined: Mar 25, 2011
|
#40039
Hi J,

Thanks for the question! First, your hypothetical has an issue since the question stem stipulates that "K is in the same group as N," and yours separates the two.

Second but equally importantly, keep in mind that this is a Must Be True question, so it's not whether M could be in Group 1, it's whether M must be in group 1. And, as it turns out, M can be in Group 2, so while (E) is possible, it is not something that has to occur.

Here's a hypothetical showing M in Group 2:

..... ..... 1: G - F - J - L
..... ..... 2: M - K - N - H

Please let me know if that helps. Thanks!

Get the most out of your LSAT Prep Plus subscription.

Analyze and track your performance with our Testing and Analytics Package.